Q8

 
bethea2010
Thanks Received: 0
Forum Guests
 
Posts: 1
Joined: May 03rd, 2010
 
 
 

PT #58, Section #3, Game #2, Question #8

by bethea2010 Mon May 03, 2010 11:02 am

Can some one please walk me through this problem. I walked through the problem and can not understand how they arrived at there answer.

Question: If Veena volunteers, then which one of the following could be true?

My answer was A. The solutions revealed the answer was B.

One of the conditions is: if S does not volunteer than V volunteers, so how would this allow B to be the correct answer?
User avatar
 
bbirdwell
Thanks Received: 864
Atticus Finch
Atticus Finch
 
Posts: 803
Joined: April 16th, 2009
 
 
 

Re: PT #58, Section #3, Game #2, Question #8

by bbirdwell Mon May 03, 2010 9:47 pm

You're right about the original constraint; it says "if NO S --> V"

You can only read those constraints from left to right. You've made the common mistake of trying to read it from right to left.

What do you know if S does NOT volunteer? That V does.

What do you know about V if S does volunteer? Nothing!

What do you know about S if V volunteers? Nothing!

Now, here's one for you: What do you know about S if V does NOT volunteer?
I host free online workshop/Q&A sessions called Zen and the Art of LSAT. You can find upcoming dates here: http://www.manhattanlsat.com/zen-and-the-art.cfm
 
MarisaP485
Thanks Received: 0
Vinny Gambini
Vinny Gambini
 
Posts: 3
Joined: October 13th, 2020
 
 
 

Re: Q8

by MarisaP485 Sun Feb 28, 2021 7:29 am

Following up here, would someone mind explaining why B is correct? I chose C. I get the response so far, but it'd be helpful to see the rest.

Thanks!
 
Laura Damone
Thanks Received: 94
Atticus Finch
Atticus Finch
 
Posts: 468
Joined: February 17th, 2011
 
 
 

Re: Q8

by Laura Damone Tue Mar 02, 2021 5:22 pm

Hi there!

For this game, the Logic Chain is the most helpful Master Diagram. For individual questions, a T-chart is the way to go. This question tells us V is in, so I draw a T chart and put V on the "In" side:

In    |  Out
V     | 

Following the logic chain, I see that V in forces T out (rule 5), so I add T to the Out group. T out forces M out (rule 2), and M out forces R out (rule 1). R out forces L in (rule 4). F and S aren't impacted, so I put them beneath the dividing line of the T chart and circle them.

So, my final T chart looks like this:

In    |  Out
V L  | T M R
     (F)
     (S)


Answers A, C, D and E all contain an element that is guaranteed to be out. Only B is possible.

Hope this helps!
Laura Damone
LSAT Content & Curriculum Lead | Manhattan Prep